0
$\begingroup$

With Matlab, I want to verify numerically whether the following inequality holds

$$\exp(-27^2)\text{erfi}(27)<\frac{21}{1000}.$$

However, I obtain "Inf" with Matlab. The reason is due to overflow, which lead to results too large to represent as conventional floating-point values.

My question is: Is there some trick to compute the LHS with Matlab?

Thanks for any help!

$\endgroup$

1 Answer 1

1
$\begingroup$

So we want to prove that $$ \frac{2}{\sqrt{\pi}}\int_{0}^{27} e^{x^2-27^2}\,dx < \frac{21}{1000}\tag{1} $$ but plenty of continued fraction representations for the Dawson function are already known.
For instance, for any $x\geq 2$ $$ \text{erfi}(x)e^{-x^2} \leq \frac{2x}{\sqrt{\pi}}\cdot\frac{1}{1+2x^2-\frac{4x^2}{2x^2-1}}\tag{2} $$ holds, and the evaluation at $x=27$ leads to $$ \frac{2}{\sqrt{\pi}}\int_{0}^{27} e^{x^2-27^2}\,dx=\frac{2}{\sqrt{\pi}}\int_{0}^{27}e^{x^2-54x}\,dx \leq 0.02092.\tag{3} $$ The middle representation is best suited for numerical purposes: $e^{x^2-54x}$ takes values $\geq 10^{-20}$ only over $[0,1]$, and $\int_{0}^{\color{red}{1}}e^{x^2-54x}\,dx $ is straightforward to approximate through Maclaurin series, convexity inequalities like $e^{x^2}\leq 1+x^2+(e-2)x^4$, Cauchy-Schwarz etc.

$\endgroup$
3
  • $\begingroup$ I did not get the part where $27^2$ becomes $54x$. Could you explain this part? $\endgroup$ Commented Nov 7, 2018 at 13:16
  • 1
    $\begingroup$ @AlexSilva: In the integral $\int_{0}^{27}e^{x^2-27^2}\,dx$ it is practical to enforce the substitution $x\mapsto 27-x$ since most of the mass of the integral is concentrated near the right endpoint. $\endgroup$ Commented Nov 7, 2018 at 13:18
  • $\begingroup$ I got it! Thanks a lot! :) $\endgroup$ Commented Nov 7, 2018 at 13:21

You must log in to answer this question.

Start asking to get answers

Find the answer to your question by asking.

Ask question

Explore related questions

See similar questions with these tags.